0 Daumen
3,7k Aufrufe

Aufgaben:

a) Sei \( a_{n}=\left(\begin{array}{c}2 n+1 \\ 3\end{array}\right) \) für alle \( n \in \mathrm{N} \).

Berechnen Sie die Summe \( S=\sum \limits_{k=1}^{3}\left(\begin{array}{l}3 \\ k\end{array}\right) a_{4 k-1} \).

b) Zeigen Sie mit vollständiger Induktion, dass folgende Formel richtig ist:

\( \sum \limits_{k=1}^{n} k(k+1)=\frac{1}{3} n(n+1)(n+2) \)


Ansatz/Problem:

Zu a)

Ich habe insgesamt 150 herausbekommen, indem ich das aktuelle k jeweils eingesetzt habe. Kann vielleicht jemand kurz sagen, ob das Ergebnis stimmt? Wenn nicht, kann ich auch meinen Rechenweg posten.

Bei b) weiß ich leider nicht, wie man vorgehen sollte, also es lässt sich ja keine Zahl für n einsetzen, nur für k. Wie komme ich jetzt hier auf das richtige Ergebnis?

Avatar von

1 Antwort

0 Daumen

a)

∑(COMB(3, k)·COMB(2·(4·k - 1) + 1, 3), k, 1, 3) = 3241

b) Zeige das es für n = 1 gilt. Dann ist das die Summe für k = 1 bis 1 also nur für k = 1

Zeige dann das wenn es für n gilt es auch für n + 1 gilt.


Behauptung:

∑ (k = 1 bis n) (k·(k + 1)) = 1/3·n·(n + 1)·(n + 2)

Induktionsanfang: n = 1

∑ (k = 1 bis n) (k·(k + 1)) = 1/3·n·(n + 1)·(n + 2)

(1·(1 + 1)) = 1/3·1·(1 + 1)·(1 + 2)

2 = 2

Induktionsschritt: n → n + 1

∑ (k = 1 bis n + 1) (k·(k + 1)) = 1/3·(n + 1)·(n + 2)·(n + 3)

∑ (k = 1 bis n) (k·(k + 1)) + (n + 1)·(n + 2) = 1/3·(n + 1)·(n + 2)·(n + 3)

1/3·n·(n + 1)·(n + 2) + (n + 1)·(n + 2) = 1/3·(n + 1)·(n + 2)·(n + 3)

1/3·n + 1 = 1/3·(n + 3)

1/3·n + 1 = 1/3·n + 1


w.z.b.w.

Avatar von 479 k 🚀

a)


Was bedeutet COMB?

Ich habe das doch so gerechnet:

(3, k) * (2 * (4k - 1) + 1) = x

(7, 3) * (3, 1) = 24

(15, 3) * (3, 2) = 51

(23, 3) * (3, 3) = 75

= 150

COMB(n, k) ist der Binomialkoeffizient (n über k)

(3, k) * (2 * (4k - 1) + 1)

(3, 1) * (7, 3) = 3 * 35 = 105

(3, 2) * (15, 3) = ...

(3, 3) * (23, 3) = ...

Wie hast du den den Binomialkoeffizienten ausgerechnet?

Ach ich glaube du hast das nicht als Binomialkoeffizienten gesehen sondern als Vektoren. Tja. Dann solltet ihr erstmal klären was dort gemeint ist.

Ein anderes Problem?

Stell deine Frage

Willkommen bei der Mathelounge! Stell deine Frage einfach und kostenlos

x
Made by a lovely community